1993 AJHSME Problems/Problem 16

Revision as of 20:09, 19 August 2011 by Mrdavid445 (talk | contribs) (Created page with "==Problem== <math> \frac{1}{1+\frac{1}{2+\frac{1}{3}}}= </math> <math> \text{(A)}\ 81.0\qquad\text{(B)}\ 82.7\qquad\text{(C)}\ 83.0\qquad\text{(D)}\ 84.0\qquad\text{(E)}\ 84.3 ...")
(diff) ← Older revision | Latest revision (diff) | Newer revision → (diff)

Problem

$\frac{1}{1+\frac{1}{2+\frac{1}{3}}}=$

$\text{(A)}\ 81.0\qquad\text{(B)}\ 82.7\qquad\text{(C)}\ 83.0\qquad\text{(D)}\ 84.0\qquad\text{(E)}\ 84.3$